LSAT and Law School Admissions Forum

Get expert LSAT preparation and law school admissions advice from PowerScore Test Preparation.

 Blueballoon5%
  • Posts: 156
  • Joined: Jul 13, 2015
|
#38241
Hi! I have a question about the main point of this reading passage. The answer key states that choice A is correct. However, doesn't it seem like this choice is too strong? I don't feel that the author makes such a strong case that "Grey marketing is unfair to trademark owners and should be legally controlled." Instead, I thought the author was simply describing the practice and one desirable solution (which wasn't even strongly worded, with only one sentence in the end indicating the author's point of view).

I chose D as the answer. Could someone explain to me why this is wrong?

Thanks so much!!!
User avatar
 Dave Killoran
PowerScore Staff
  • PowerScore Staff
  • Posts: 5862
  • Joined: Mar 25, 2011
|
#38247
Hi Blue,

The first reference here would be for our passage overview, at: lsat/viewtopic.php?t=11038

The passage as a whole goes through a process of defining a problem, then showing how one group is hurt by the practice ("trademark owners"), before discussing several theories of trademark law. The passage then wraps up by saying that the theory that gives trademark owners the most protection is the one the author expects to prevail. In other words, the author shows us a problem, shows us who it hurts, and then sums up by saying that the theory that controls it best should be and will be the one used.

With answer choice (D), this is something that occurred as part of the discussion, but is that the main point? I wouldn't say so, primarily because it only addresses what happens in the last paragraph. In other words, the main point is more than just what is stated in (D).

On the other hand, (A) captures more of the passage, including the author's final, strong statement of future direction. By more broadly encompassing the ideas in the passage, (A) is a strong contender.

Note about the first two paragraphs: when you see an author setting up context and explaining what's going on, that's typically background info. It's like the premises in an LR problem. That's not the main point, but rather what they build on those premises/context is the main point. (A) captures that better than (D), hence (A) is correct.

Please let me know if that helps. Thanks!
 Blueballoon5%
  • Posts: 156
  • Joined: Jul 13, 2015
|
#38250
Thanks so much! I found the link to be very helpful as well. In the future (with reading comp passages), I will try to find these links on the forum to better help me understand the passage.

I have one question. I noticed that the link (lsat/viewtopic.php?t=11038) to the reading comp post is listed under "PrepTest 8 - June 1993 - LSAT Answers and Explanations - RC," and not "Page 2-26" (from the lesson books). Do you know if there is an easier way for me to find these posts in the future as I go through the lesson books? E.g. how to find the post for the reading passage on page 2-28.

Thanks so much!!!
User avatar
 Dave Killoran
PowerScore Staff
  • PowerScore Staff
  • Posts: 5862
  • Joined: Mar 25, 2011
|
#38255
Great, glad that helped! Those cross-links for each problem to the Forum explanation should be in your Online Student Center—we had added all those about a year or two ago, so if you don't see them in the individual lesson portions, just let us know.

Thanks!

Get the most out of your LSAT Prep Plus subscription.

Analyze and track your performance with our Testing and Analytics Package.